Proving a nonempty finite subset of a group is a subgroup

Click For Summary
To prove that a nonempty finite subset H of a group G is a subgroup, it is essential to show that H contains the identity element, is closed under the group operation, and includes inverses for all its elements. Given that H is finite and closed under the operation, if H contains only one element, that element must be the identity. For larger subsets, the finiteness implies that elements must repeat, leading to the conclusion that the identity is present and every element has an inverse within H. Thus, H satisfies all subgroup criteria, confirming that H is indeed a subgroup of G.
eq1
Messages
206
Reaction score
71

Homework Statement



Let a non empty finite subset H of a group G be closed under the binary operation * on G. Show that H is a subgroup of H.

2. Relevant Definitions

Group Properties:
G1: a*(b*c)=(a*b)*c for all a,b,c in G
G2: e*x=x*e=x for all x in G
G2: if x is in G then x' is in G and x*x'=x'*x=e

A subset H of group G is a subgroup of G iff:
S1: the identity element of G, e, is in H
S2: the binary operation on G is closed on H
S3: if x is in H, then x' is in H and x*x'=x'*x=e


The Attempt at a Solution



So I know

H != {}
H is finite
H is closed, so I get the second property of a subgroup for free.
H is subset of the elements of G and inherits the operation.

If H only has one element, I only need to show that that element is the identity of G. I think I can do this pretty easy with the closure property.

If H has more then one element here is my strategy,

Assume x is in H and for every y in H x*y != e.
This breaks associativity on G which is a contraction since G is a group.
Therefore given x in H for some y in H x*y=e which means y is x's inverse and since it
is in H and since H is closed e is there too.

The reason I think this is the way to go is because I drew tables for a couple of binary operations for a small set that was a group. Then I added some elements around it with a new pseudo-identity element over the larger wanna-be group. Associativity broke on the larger operation, which means it couldn't have been a group.

So it seems like I should be able to sketch a proof around this concept. Given H is closed and associative and assume it does not contain the identity of G. This breaks associativity of G which is a contradiction. But I am not really getting anywhere.

Is this approach even valid? I am starting to suspect not. I am pretty stuck. Could someone perhaps suggest another direction? I am not really taking advantage of the finite piece of information. Maybe there is something there?
 
Physics news on Phys.org
Suppose x is in H. Since H is closed under *, then x^n = x*x*...*x is in H for all n > 0.

But H is finite, so ...
 
Right, of course.

I think I have an outline of a proof that works.

H={x^n,for every n in Z}
Proof:
Assume there is a y in H where y != x^i. I can show this is a contradiction
by knowing x*c=y has a solution in G, and c cannot be a power of x, or y is a power
of x. But x^t*y is in H due to closure and for some t, x^t*y=e otherwise H is not
finite. But then x^t*y = x^t*x*c = e.
A contradiction, so c is a power of x, and y = x^i.

So now I know H is finite and has a generator x.
So x^0=e
And if |H|=n then x^i*x^(n-i)=e so there is an inverse for every element

H is a subgroup of G

I probably made that way harder than is had to be.
 
Last edited:
Why should it follow that x^t*y=e?

H certainly doesn't have to be cyclic. The purpose of my hint above was to let you invoke the finiteness of H to see that not all the x^n's are distinct, i.e. there must be an s and a t with, say, s>t, such that x^s = x^t. From this, one can deduce that e is in H and that x is invertible. But since x was arbitrary, this means every element has an inverse in H, so it's a subgroup of G.
 
Question: A clock's minute hand has length 4 and its hour hand has length 3. What is the distance between the tips at the moment when it is increasing most rapidly?(Putnam Exam Question) Answer: Making assumption that both the hands moves at constant angular velocities, the answer is ## \sqrt{7} .## But don't you think this assumption is somewhat doubtful and wrong?

Similar threads

Replies
2
Views
2K
  • · Replies 7 ·
Replies
7
Views
2K
  • · Replies 5 ·
Replies
5
Views
2K
  • · Replies 9 ·
Replies
9
Views
2K
  • · Replies 3 ·
Replies
3
Views
2K
  • · Replies 2 ·
Replies
2
Views
2K
  • · Replies 3 ·
Replies
3
Views
2K
  • · Replies 1 ·
Replies
1
Views
1K
  • · Replies 5 ·
Replies
5
Views
2K
  • · Replies 1 ·
Replies
1
Views
2K